Mathcenter Forum  

Go Back   Mathcenter Forum > คณิตศาสตร์โอลิมปิก และอุดมศึกษา > ข้อสอบโอลิมปิก
สมัครสมาชิก คู่มือการใช้ รายชื่อสมาชิก ปฏิทิน ข้อความวันนี้

ตั้งหัวข้อใหม่ Reply
 
เครื่องมือของหัวข้อ ค้นหาในหัวข้อนี้
  #1  
Old 29 เมษายน 2010, 07:13
Switchgear's Avatar
Switchgear Switchgear ไม่อยู่ในระบบ
บัณฑิตฟ้า
 
วันที่สมัครสมาชิก: 12 มกราคม 2006
ข้อความ: 472
Switchgear is on a distinguished road
Default เฉลยหนังสือ สอวน. "อสมการและสมการเชิงฟังก์ชัน" (ส่วนที่ 1 อสมการ)

ช่วงนี้น้องๆ คนเก่งหลายคนกำลังเข้าค่าย 2 สอวน. อยู่ น่าจะมีคนติดโจทย์
ในหนังสือ "อสมการและสมการเชิงฟังก์ชัน" ของ สอวน. หลายคน รวมทั้ง
น้องๆ ที่กำลังเตรียมตัวสอบคัดเลือกเข้าค่าย สอวน. ปีต่อไป

ผมเลยเข้ามาชวนให้ช่วยกันเฉลยโจทย์ในหนังสือ "อสมการและสมการเชิงฟังก์ชัน"
ของ สอวน. (ส่วนที่ 1 อสมการ) ในกระทู้นี้

สำหรับส่วนที่ 2 ผมจะแยกไปอีกกระทู้หนึ่ง ไม่งั้นปวดหัวแย่ ... เวลาค้นหาทีหลัง

หมายเหตุ:
1. แบบฝึกหัดถูกแยกอยู่ท้ายหัวข้อย่อย ไม่มีแบบฝึกหัดท้ายบทโดยเฉพาะ จึงมีแบบฝึกหัดเยอะมาก
2. ผมไม่ได้แยกกระทู้แต่ละบท ฉะนั้นผู้เฉลยช่วยระบุให้ชัดด้วยว่าเฉลยบทไหน ท้ายหัวข้อย่อยไหน และข้อที่เท่าไร
3. คงต้องช่วยกันทุกคนครับ ผมรู้ว่ากระทู้นี้สำเร็จยาก :-)
__________________
หนึ่งปีของอัจฉริยะ อาจเทียบเท่าชั่วชีวิตของคนบางคน

30 เมษายน 2010 18:00 : ข้อความนี้ถูกแก้ไขแล้ว 6 ครั้ง, ครั้งล่าสุดโดยคุณ Switchgear
ตอบพร้อมอ้างอิงข้อความนี้
  #2  
Old 30 เมษายน 2010, 05:51
Switchgear's Avatar
Switchgear Switchgear ไม่อยู่ในระบบ
บัณฑิตฟ้า
 
วันที่สมัครสมาชิก: 12 มกราคม 2006
ข้อความ: 472
Switchgear is on a distinguished road
Default

ผมจะโพสต์โจทย์ไว้เพื่อความสะดวก เผื่อว่าบางคนอาจสนใจช่วยเฉลย แต่ไม่มีตัวเล่ม (หรือเหตุผลอื่น...)

บทที่ 1: พื้นฐานของอสมการ

แบบฝึกหัดท้ายบทที่ 1 (มีทั้งหมด 15 ข้อ; บทนี้ไม่ได้แยกแบบฝึกหัดไปอยู่ในหัวข้อย่อย)

1. กำหนดให้ $a, b, c \in R^{+}\;$ โดยที่ $a+b+c \geqslant abc\;$ จงแสดงว่า $\;a^2+b^2+c^2 \geqslant abc$

2. กำหนดให้ $a, b \in R^{+}\;$ โดยที่ $a \not= b\;$ จงพิสูจน์ว่า $\;\displaystyle{\frac{a}{b^2}+\frac{b}{a^2} > \frac{1}{a} + \frac{1}{b}}$

3. จงพิสูจน์ว่า $\;\displaystyle{\frac{a^2+2}{\sqrt{a^2+1}} \geqslant 2}\;$ สำหรับทุกจำนวนจริง $a$

4. กำหนดให้ $a, b, c \in R\;$ โดยที่ $\;a^2+b^2+c^2 = 1\;$ จงแสดงว่า $\;-\frac{1}{2} \leqslant ab+bc+ca \leqslant 1$
$\quad$(เดิมผมโพสต์โจทย์ผิด ผมแก้จาก 1/2 เป็น -1/2 ตามที่มีผู้แนะนำไว้แล้วครับ ... ขอบคุณมาก!)

5. จงพิสูจน์ว่า $\;\displaystyle{\frac{1}{15} < \frac{1}{2}\cdot \frac{3}{4}\cdot \frac{5}{6}\cdot \frac{7}{8}\cdot \cdot \cdot \frac{99}{100} < \frac{1}{10}}$

6. ให้ $\;\displaystyle{Q_n = 1 + \frac{1}{4} + \frac{1}{9} + ... + \frac{1}{n^2}}\;$ จงพิสูจน์ว่า $\;\displaystyle{\frac{19}{12} < Q_n < \frac{7}{4} - \frac{1}{n}\qquad (n \geqslant 3)}$

7. ให้ $\;n > 1\;$ เป็นจำนวนนับ จงพิสูจน์ว่า $\;\displaystyle{\frac{1}{2} < \frac{1}{n+1} + \frac{1}{n+2} + ... + \frac{1}{2n} < \frac{3}{4}}$

8. เมื่อ $a, b \in R_0 \;$ จงพิสูจน์ว่า $a^3(b+1) + b^3(a+1) \geqslant a^2(b+b^2) + b^2(a+a^2)$

9. จงพิสูจน์ว่า $11\cdot\sqrt[1994]{10} > 10 + \sqrt[1000]{10}$

10. ถ้า $a, b \in R^{+}\;$ แล้ว จงพิสูจน์ว่า $\;\displaystyle{\frac{1}{a}+\frac{4}{b} > \frac{9}{a+b}}$

11. จงพิสูจน์ว่า $\left(\,1-\frac{1}{(1993)^3}\right) \left(\,1-\frac{1}{(1994)^3}\right) ... \left(\,1-\frac{1}{n^3} \right) > \frac{1992}{1993}\;$ เมื่อ $n$ เป็นจำนวนนับ $\geqslant 1993$

12. จงแก้อสมการ $(x^3+x^2+1)^2 > 4x^3(x-1)^2$

13. ให้ $\;a_1,...,a_n (n\geqslant 2)\;$ เป็นจำนวนเต็ม โดยที่ $1 \leqslant a_1 \leqslant ... \leqslant a_n\;$
$\quad\;\;$ จงแสดงว่าอสมการ $\; \sum_{i = 1}^{n} a_i x_i^2 + 2 \sum_{i = 1}^{n-1} x_i x_{i+1} > 0\;$ เป็นจริง สำหรับทุก $n-$อันดับ
$\quad\;\;$ ของจำนวนจริง $\;(x_1,...,x_n) \not= (0,...,0)\;$ ก็ต่อเมื่อ $\; a_2 \geqslant 2$

14. จงหาจำนวนนับ $n$ ที่ทำให้อสมการต่อไปนี้เป็นจริง
$\quad\;\;$ สำหรับทุก $n-$อันดับของจำนวนจริงบวก $\;(a_1,...,a_n): \sum_{i = 1}^{n} a_i^2 \times \sum_{i = 1}^{n} a_i - \sum_{i = 1}^{n} a_i^3 \geqslant 6\prod_{i = 1}^{n} a_i$

15. ให้ $\; m, n \in N\;$ จงพิสูจน์ว่า $\;\displaystyle{\left|\,\frac{m+2n}{m+n} - \sqrt{2} \right| < \;\left|\,\frac{m}{n} - \sqrt{2} \right|}$

สำหรับโจทย์ของบทที่ 2 ผมจะโพสต์ไว้ที่ความเห็น #15 ... ขอบคุณที่ติดตาม :-)
__________________
หนึ่งปีของอัจฉริยะ อาจเทียบเท่าชั่วชีวิตของคนบางคน

11 พฤษภาคม 2010 18:57 : ข้อความนี้ถูกแก้ไขแล้ว 9 ครั้ง, ครั้งล่าสุดโดยคุณ Switchgear
ตอบพร้อมอ้างอิงข้อความนี้
  #3  
Old 30 เมษายน 2010, 08:31
nooonuii nooonuii ไม่อยู่ในระบบ
ผู้พิทักษ์กฎทั่วไป
 
วันที่สมัครสมาชิก: 25 พฤษภาคม 2001
ข้อความ: 6,408
nooonuii is on a distinguished road
Default

บทที่ 1 ให้ใช้อะไรได้บ้างครับ ใช้แค่อสมการพื้นฐานหรือเปล่า
__________________
site:mathcenter.net คำค้น
ตอบพร้อมอ้างอิงข้อความนี้
  #4  
Old 30 เมษายน 2010, 09:49
กระบี่เดียวดายแสวงพ่าย's Avatar
กระบี่ประสานใจ
 
วันที่สมัครสมาชิก: 21 กุมภาพันธ์ 2009
ข้อความ: 647
กระบี่เดียวดายแสวงพ่าย is on a distinguished road
Default

งั้นผมเริ่มจากข้อ 2. ก็แล้วกันครับ
$(a-b)^2>0$ เพราะ $a\not= b$
$a^2-2ab+b^2>0$
$a^-ab+b^2>ab$
$(a+b)(a^-ab+b^2)>(a+b)ab$
$a^3+b^3>(a+b)ab$
$\frac{a^3+b^3}{a^2b^2} >\frac{(a+b)ab}{a^2b^2}$
$\frac{a}{b^2} +\frac{b}{a^2} >\frac{1}{a}+\frac{1}{b}$
ตอบพร้อมอ้างอิงข้อความนี้
  #5  
Old 30 เมษายน 2010, 09:59
Ne[S]zA's Avatar
Ne[S]zA Ne[S]zA ไม่อยู่ในระบบ
ลมปราณไร้สภาพ
 
วันที่สมัครสมาชิก: 13 กรกฎาคม 2008
ข้อความ: 1,221
Ne[S]zA is on a distinguished road
Default

ข้อ3.โดย AM.-GM. Inequality จะได้
$\dfrac{a^2+1+1}{2}\geqslant\sqrt{a^2+1}$
นั่นคือ $\dfrac{a^2+2}{\sqrt{a^2+1}} \geqslant 2$
__________________
||!<<<<iNesZaii>>>>!||

30 เมษายน 2010 09:59 : ข้อความนี้ถูกแก้ไขแล้ว 1 ครั้ง, ครั้งล่าสุดโดยคุณ Ne[S]zA
ตอบพร้อมอ้างอิงข้อความนี้
  #6  
Old 30 เมษายน 2010, 10:50
กระบี่เดียวดายแสวงพ่าย's Avatar
กระบี่ประสานใจ
 
วันที่สมัครสมาชิก: 21 กุมภาพันธ์ 2009
ข้อความ: 647
กระบี่เดียวดายแสวงพ่าย is on a distinguished road
Default

ข้อ.3 แบบธรรมดาๆ ครับ
$a^4\geqslant 0$
$a^4+4a^2+4\geqslant 4a^2+4$
$(a^2+2)^2\geqslant 4(a^2+1)$
$a^2+2\geqslant 2\sqrt{a^2+1} $
$\frac{a^2+2}{2} \geqslant \sqrt{a^2+1} $
$\frac{a^2+2}{\sqrt{a^2+1}} \geqslant 2 $
ตอบพร้อมอ้างอิงข้อความนี้
  #7  
Old 30 เมษายน 2010, 17:46
Switchgear's Avatar
Switchgear Switchgear ไม่อยู่ในระบบ
บัณฑิตฟ้า
 
วันที่สมัครสมาชิก: 12 มกราคม 2006
ข้อความ: 472
Switchgear is on a distinguished road
Default

อ้างอิง:
ข้อความเดิมเขียนโดยคุณ nooonuii View Post
บทที่ 1 ให้ใช้อะไรได้บ้างครับ ใช้แค่อสมการพื้นฐานหรือเปล่า
เดิมผมเข้าใจผิดว่าแบบฝึกหัดตอนท้ายของแต่ละบทนั้น เป็นแบบฝึกหัดรวม แต่ปรากฎว่ามันเป็น
แบบฝึกหัดของหัวข้อสุดท้ายเท่านั้น นั่นคือ เขาแยกโจทย์ที่มีเยอะมาก ไปอยู่ท้ายหัวข้อที่เกี่ยวข้อง
ฉะนั้นตามโจทย์ที่แก้ไขใหม่ ผมจะระบุไว้ว่าเป็นโจทย์ของหัวข้ออะไร ? จะได้เลือกเทคนิคได้ตรงขึ้น

สำหรับบทที่ 1 เขาไม่ได้ตั้งชื่อหัวข้อย่อยเลย ฉะนั้นก็ใช้อสมการพื้นฐานเป็นหลัก แต่ถ้าใครมีเทคนิค
ชั้นสูงที่ปราบโจทย์ได้เจ๋งๆ ก็ช่วยโพสต์ไว้ด้วยครับ เพื่อให้ผู้อ่านได้รับประโยชน์เพิ่มขึ้นอีก :-)
__________________
หนึ่งปีของอัจฉริยะ อาจเทียบเท่าชั่วชีวิตของคนบางคน

30 เมษายน 2010 18:04 : ข้อความนี้ถูกแก้ไขแล้ว 1 ครั้ง, ครั้งล่าสุดโดยคุณ Switchgear
ตอบพร้อมอ้างอิงข้อความนี้
  #8  
Old 30 เมษายน 2010, 20:47
nooonuii nooonuii ไม่อยู่ในระบบ
ผู้พิทักษ์กฎทั่วไป
 
วันที่สมัครสมาชิก: 25 พฤษภาคม 2001
ข้อความ: 6,408
nooonuii is on a distinguished road
Default

อ้างอิง:
ข้อความเดิมเขียนโดยคุณ Switchgear View Post
1. กำหนดให้ $a, b, c \in R^{+}\;$ โดยที่ $a+b+c \geqslant abc\;$ จงแสดงว่า $\;a^2+b^2+c^2 \geqslant abc$
$(a^2+b^2+c^2)^2\geq (ab+bc+ca)^2\geq 3abc(a+b+c)\geq 3(abc)^2$
__________________
site:mathcenter.net คำค้น
ตอบพร้อมอ้างอิงข้อความนี้
  #9  
Old 30 เมษายน 2010, 23:50
nooonuii nooonuii ไม่อยู่ในระบบ
ผู้พิทักษ์กฎทั่วไป
 
วันที่สมัครสมาชิก: 25 พฤษภาคม 2001
ข้อความ: 6,408
nooonuii is on a distinguished road
Default

อ้างอิง:
ข้อความเดิมเขียนโดยคุณ Switchgear View Post
4. กำหนดให้ $a, b, c \in R^{+}\;$ โดยที่ $\;a^2+b^2+c^2 = 1\;$ จงแสดงว่า $\;\frac{1}{2} \leqslant ab+bc+ca \leqslant 1$
$\bullet ab+bc+ca\leq a^2+b^2+c^2=1$

$\bullet \dfrac{1}{2}\not\leq ab+bc+ca$

ตัวอย่าง : $a=\dfrac{1}{10},b=\dfrac{1}{10},c=\dfrac{\sqrt{98}}{10}$

$\bullet$ ถ้า $a,b,c$ เป็นความยาวด้านของรูปสามเหลี่ยมแล้ว

$\dfrac{1}{2}< ab+bc+ca$

พิสูจน์ : $a^2+b^2+c^2<2(ab+bc+ca)\Leftrightarrow a(b+c-a)+b(c+a-b)+c(a+b-c)>0$
__________________
site:mathcenter.net คำค้น

01 พฤษภาคม 2010 09:26 : ข้อความนี้ถูกแก้ไขแล้ว 2 ครั้ง, ครั้งล่าสุดโดยคุณ nooonuii
ตอบพร้อมอ้างอิงข้อความนี้
  #10  
Old 01 พฤษภาคม 2010, 07:00
Switchgear's Avatar
Switchgear Switchgear ไม่อยู่ในระบบ
บัณฑิตฟ้า
 
วันที่สมัครสมาชิก: 12 มกราคม 2006
ข้อความ: 472
Switchgear is on a distinguished road
Default

ขอบคุณครับที่ช่วยกันโพสต์เฉลย ผมเอาโจทย์บทที่ 1 มาเพิ่มถึงข้อ 9 แล้ว
ถ้ากระทู้นี้มีเฉลยเยอะพอ ต่อไปน่าจะเป็นแหล่งข้อมูลให้เด็กรุ่นใหม่ใช้ค้นคว้า
ในการเตรียมสอบ สอวน. ในส่วนนี้อย่างเต็มที่

แต่คงไม่ใช่กระทู้ที่สำเร็จง่ายๆ เพราะโจทย์เยอะจริงๆ :-)
__________________
หนึ่งปีของอัจฉริยะ อาจเทียบเท่าชั่วชีวิตของคนบางคน
ตอบพร้อมอ้างอิงข้อความนี้
  #11  
Old 01 พฤษภาคม 2010, 09:43
Keehlzver's Avatar
Keehlzver Keehlzver ไม่อยู่ในระบบ
บัณฑิตฟ้า
 
วันที่สมัครสมาชิก: 26 มกราคม 2009
ข้อความ: 533
Keehlzver is on a distinguished road
Default

ก่อนอื่นต้องขอขอบคุณจริงๆที่มีกระทู้ดีๆเเบบนี้นะครับ เพราะผมเองก็อยากได้เฉลยอยู่เหมือนกัน
ข้อ $4$ จริงๆ โจทย์ในหนังสือเป็นเเบบนี้นะครับ $a,b,c\in \mathbb{R}$ โดยที่ $a^2+b^2+c^2=1$ จงเเสดงว่า $-\frac{1}{2}\leq ab+bc+ca \leq 1$ เเต่ version ที่พี่ Nooonuii ทำก็สุดยอดเหมือนกันเลยได้โจทย์ใหม่มาอีก $1$ ข้อเป็นความรู้ไว้ ผมอยากจะบอกก่อนว่าข้อ 5 ผมได้เเนวคิดมาจากกระทู้เก่าของคุณสุธีครับ

ข้อ $4$ $a,b,c\in \mathbb{R}$ โดยที่ $a^2+b^2+c^2=1$ จงเเสดงว่า $-\frac{1}{2}\leq ab+bc+ca \leq 1$


ข้อ $5$ จงพิสูจน์ว่า $\frac{1}{15}<\frac{1}{2}\frac{3}{4}\frac{5}{6}\frac{7}{8}\cdots\frac{99}{100}<\frac{1}{10}$


ข้อ $6$ ให้ $Q_n=1+\frac{1}{4}+\frac{1}{9}+\cdots\frac{1}{n^2}$ จงพิสูจน์ว่า $\frac{19}{12}-\frac{1}{n+1}<Q_n<\frac{7}{4}-\frac{1}{n}$


ข้อ $7$ ให้ $n>1$ เป็นจำนวนนับ จงพิสูจน์ว่า $\frac{1}{2}<\frac{1}{n+1}+\frac{1}{n+2}+\cdots+\frac{1}{2n}<\frac{3}{4}$


ข้อ $8$ เมื่อ $a,b$ เป็นจำนวนจริงที่ไม่ติดลบ จงพิสูจน์ว่า $a^3(b+1)+b^3(a+1)\geq a^2(b+b^2)+b^2(a+a^2)$


ข้อ $9$ จงพิสูจน์ว่า $11\cdot\sqrt[1994]{10}>10+\sqrt[1000]{10}$


มีตรงไหนผิดหรือควรจะเขียนอย่างไรก็ติเลยนะครับ

ข้อ $7$ อสมการข้างขวาเราต้องพิสูจน์ว่า $\frac{1}{k+1}+\frac{1}{k+2}+\cdots+\frac{1}{2k}+\frac{1}{2k+1}+\frac{1}{2k+2}<\frac{3}{4}$ เมื่อ $k\geq 2$ เเต่พอลองเเทนลงไปเเล้วอสมการเป็นเท็จ เเบบนี้หมายความว่าพิสูจน์โดยอุปนัยไม่ได้เหรอครับ
__________________
"ชั่วโมงหน้าต้องดีกว่าเดิม!"

01 พฤษภาคม 2010 14:25 : ข้อความนี้ถูกแก้ไขแล้ว 1 ครั้ง, ครั้งล่าสุดโดยคุณ Keehlzver
ตอบพร้อมอ้างอิงข้อความนี้
  #12  
Old 01 พฤษภาคม 2010, 10:59
nooonuii nooonuii ไม่อยู่ในระบบ
ผู้พิทักษ์กฎทั่วไป
 
วันที่สมัครสมาชิก: 25 พฤษภาคม 2001
ข้อความ: 6,408
nooonuii is on a distinguished road
Default

อ้างอิง:
ข้อความเดิมเขียนโดยคุณ Switchgear View Post
5. จงพิสูจน์ว่า $\;\frac{1}{15} < \frac{1}{2}\cdot \frac{3}{4}\cdot \frac{5}{6}\cdot \frac{7}{8}\cdot \cdot \cdot \frac{99}{100} < \frac{1}{10}$
เพิ่ม generalization ให้

$\dfrac{1}{2\sqrt{n}}\leq \dfrac{1}{2}\cdot \dfrac{3}{4}\cdots \dfrac{2n-1}{2n}\leq\dfrac{1}{\sqrt{3n+1}}$

พิสูจน์โดยใช้อุปนัยเชิงคณิตศาสตร์ครับ
__________________
site:mathcenter.net คำค้น
ตอบพร้อมอ้างอิงข้อความนี้
  #13  
Old 01 พฤษภาคม 2010, 11:02
nooonuii nooonuii ไม่อยู่ในระบบ
ผู้พิทักษ์กฎทั่วไป
 
วันที่สมัครสมาชิก: 25 พฤษภาคม 2001
ข้อความ: 6,408
nooonuii is on a distinguished road
Default

ข้อ $7$ ยังมีปัญหาตรงอสมการข้างขวาครับ ลองดูใหม่อีกครั้ง
__________________
site:mathcenter.net คำค้น
ตอบพร้อมอ้างอิงข้อความนี้
  #14  
Old 01 พฤษภาคม 2010, 13:32
Little Penguin Little Penguin ไม่อยู่ในระบบ
จอมยุทธ์หน้าใหม่
 
วันที่สมัครสมาชิก: 11 ตุลาคม 2009
ข้อความ: 65
Little Penguin is on a distinguished road
Default

ข้อ 4 ผมเชื่อว่าอสมการฝั่งซ้ายเป็น มากกว่าหรือเท่ากับ $-\dfrac{1}{2}$

ข้อ 7 ฝั่งขวา
ตอบพร้อมอ้างอิงข้อความนี้
  #15  
Old 01 พฤษภาคม 2010, 13:53
Switchgear's Avatar
Switchgear Switchgear ไม่อยู่ในระบบ
บัณฑิตฟ้า
 
วันที่สมัครสมาชิก: 12 มกราคม 2006
ข้อความ: 472
Switchgear is on a distinguished road
Default

ข้อ 4 ผมพิมพ์โจทย์ผิดจริงๆ ครับ ต้องเป็น -1/2 ตามที่ติงไว้ ... ขอบคุณมากครับ

ดีใจมากที่มีคนช่วยโพสต์เฉลยเยอะๆ ค่อยดูมีความหวังจะให้เป็นกระทู้รวบรวมหน่อย

#11 คุณ Keehlzver ช่วยโพสต์เยอะดี ไม่ทราบว่ากระทู้เก่าของคุณสุธีนั้น เขาเฉลย
ไว้ครบอยู่แล้วหรือเปล่า ? ผมจะได้ไม่ต้องทำซ้ำ !

แต่ถ้ามีหลายข้อที่เฉลยแนวคิดต่างออกไป ก็จะเป็นประโยชน์กับผู้เยี่ยมชมมากขึ้น !

มาเริ่มโจทย์สำหรับบทที่ 2 กันดีกว่า ...

บทที่ 2: อสมการค่าเฉลี่ยเลขคณิต-เรขาคณิต

แบบฝึกหัดท้ายหัวข้อ 2.1: อสมการ AM-GM (มี 15 ข้อ)

1. ให้ $\;a,b,c \in R_0\;$ จงพิสูจน์ว่า $(a+b)(b+c)(c+a) \geqslant 8abc$

2. ให้ $\;a_i > 0 (i = 1,...,n)\;$ สอดคล้องกับ $\;a_1a_2...a_n = 1\;$ จงพิสูจน์ว่า $\;(1+a_1)(1+a_2)...(1+a_n) \geqslant 2^n$

3. ให้ $\;a,b,c \in R_0\;$ จงแสดงว่า $\sqrt{(a+c)(b+d)} \geqslant \sqrt{ab} + \sqrt{cd}$

4. (APMC 1963) ให้ $\;a,b,x,y \in R_0\;$ สอดคล้องกับเงื่อนไข $\; a^5 + b^5 \leqslant 1, x^5 + y^5 \leqslant 1 \;$ จงแสดงว่า $a^2x^3 + b^2y^3 \leqslant 1$

5. (APMC 1971) ให้ $\;n \in N, n \geqslant 2\;$ และ $\;a, x_1,...,x_n \in R^+\;$ จงพิสูจน์ว่า
$\qquad \displaystyle{\frac{a^{x_1-x_2}}{x_1+x_2} + \frac{a^{x_2-x_3}}{x_2+x_3} + ... + \frac{a^{x_n-x_1}}{x_n+x_1} \geqslant \frac{n^2}{2(x_1+...+x_n)}}$
$\quad$ และหาเงื่อนไขในการที่จะได้สมการ

6. ให้ $\;a_1,...,a_n,b_1,...,b_n,c_1,...c_n$ เป็นจำนวนจริงบวก จงพิสูจน์ว่า
$\qquad \left(\,\sum_{i = 1}^{n} a_ib_ic_i \right)^3 \leqslant \left(\,\sum_{i = 1}^{n} a_i^3 \right)^3 + \left(\,\sum_{i = 1}^{n} b_i^3 \right)^3 + \left(\,\sum_{i = 1}^{n} c_i^3 \right)^3$

7. ให้ $\;a,b,c \in R^+$ จงพิสูจน์ว่า $\;2\sqrt{ab+bc+ca} \leqslant \sqrt{3}\; \sqrt[3]{(a+b)(b+c)(c+a)} $

8. ให้ $\;a,b,c \in R^+$ จงพิสูจน์ว่า $\; \displaystyle{\frac{a+b+c}{3} \geqslant \sqrt{\frac{ab+bc+ca}{3}}\; \sqrt[3]{abc}} $

9. ถ้า $\;a,b,c,d$ เป็นจำนวนจริงบวกแล้ว จงแสดงว่า หนึ่งในสามอสมการต่อไปนี้ไม่เป็นจริง
$\qquad a+b < c+d,\quad (a+b)(c+d) < ab+cd,\quad (a+b)cd < ab(c+d)$

10. (Putnam 1998) จงหาค่าต่ำสุดของ $\; \displaystyle{\frac{(x+\frac{1}{x})^6 - (x+\frac{1}{x^6}) - 2}{(x+\frac{1}{x})^3 + (x^3+\frac{1}{x^3})}}\;$ เมื่อ $x \in R^+$

11. (Bulgarian MC 1997) ให้ $\;a,b,c \in R^+$ มีสมบัติว่า $abc = 1\;$ จงพิสูจน์ว่า
$\qquad \displaystyle{\frac{1}{1+a+b} + \frac{1}{1+b+c} + \frac{1}{1+c+a} \leqslant \frac{1}{2+a} + \frac{1}{2+b} + \frac{1}{2+c}}$

12. (Austrian MO 2002) จงหาค่าสูงสุดของจำนวนจริง $C$ ที่ทำให้อสมการ
$\qquad \displaystyle{\frac{\{(x+y)^2-6\}\{x-y)^2+8\}}{(x-y)^2} \geqslant C}$ เป็นจริง สำหรับทุก $x,y \in R$ ที่มีสมบัติว่า $xy = 2, x \not= y$
$\qquad $และหาด้วยว่า คู่อันดับ $(x,y)$ ที่ทำให้อสมการข้างต้นเป็นสมการ สำหรับค่า $C$ ดังกล่าว

13. (Belarusian 2002) จงหาค่าต่ำสุดของ $S = a_1a_2a_3 + b_1b_2b_3 + c_1c_2c_3$ เมื่อ
$\qquad (a_1,a_2,a_3,b_1,b_2,b_3,c_1,c_2,c_3)$ เป็นการเรียงสับเปลี่ยนของจำนวนเต็ม $1,2,3,4,5,6,7,8,9$

14. ให้ $\;a_1,...,a_n \in R^+$ นิยาม $A = (a_1+...+a_n)/n, \;G = (a_1...a_n)^{1/n}, \;H = n/(a_1^{-1}+...+a_n^{-1})\;$

$\qquad (i)\quad$ ถ้า $n$ เป็นคู่ แล้ว จงแสดงว่า $\;\displaystyle{\frac{A}{H}\; \leqslant -1 + 2(\frac{A}{G})^n}$

$\qquad (ii)\quad$ ถ้า $n$ เป็นคี่ แล้ว จงแสดงว่า $\;\displaystyle{\frac{A}{H}\; \leqslant - \frac{n-2}{n} + \frac{2(n-1)}{n}(\frac{A}{G})^n}$

15. ให้ $\;x_1,...x_n,x_{n+1} \in R^+$ สอดคล้องกับ $x_1 + ... + x_n = x_{n+1}$
$\qquad $ จงพิสูจน์ว่า $\; \sum_{i = 1}^{n} \sqrt{x_1(x_{n+1} - x_i)} \;\leqslant \;\sqrt{\sum_{i = 1}^{n} x_{n+1}(x_{n+1} - x_i)}$


แบบฝึกหัดท้ายหัวข้อ 2.2: อสมการ AM-GM ถ่วงน้ำหนัก (มี 6 ข้อ)

1. (British MO) ให้ $\;p,q,r \in R_0\;$ มีสมบัติว่า $\;p+q+r = 1\;$ จงพิสูจน์ว่า $7(pq+qr+rp) \leqslant 2+9pqr$

2. ให้ $\;I,J \in N\;$ และ $\;a_{ij} \in R^+\;(1 \leqslant i \leqslant I,\; 1 \leqslant j \leqslant J,\;$
$\qquad $จงพิสูจน์ว่า ถ้า $q \geqslant p > 0\;$ แล้ว $\; \left(\,\sum_{j = 1}^{J} \left(\,\sum_{i = 1}^{I} a_{ij}^p \right) ^{q/p}\right)^{1/q} \leqslant \left(\,\sum_{i = 1}^{I} \left(\,\sum_{j = 1}^{J} a_{ij}^p \right) ^{p/q}\right)^{1/p}$

3. ให้ $\;x,y,z\;$ เป็นจำนวนจริงที่มีค่า $\geqslant 2\;$ จงพิสูจน์ว่า $(y^3+x)(z^3+y)(x^3+z) \geqslant 125xyz$

4. ให้ $\;n \in N;$ จงหาค่าต่ำสุดของนิพจน์ $\;\displaystyle{x_1 + \frac{x_2^2}{2} + \frac{x_3^3}{3} + ... + \frac{x_n^n}{n}}$
$\qquad$ เมื่อ $x_1,...,x_n\;$ แปรไปเหนือ $R^+$ และสอดคล้องกับสมการ $\;\displaystyle{\frac{1}{x_1} + \frac{1}{x_2} + ... + \frac{1}{x_n} = n}$

5. (Math. Magazine 1996) ถ้า $\;a,b,c,r \in R^+\;$ จงพิสูจน์ว่า $(abc)^{1/3} \leqslant \left(\,a^{a^r}b^{b^r}c^{c^r}\right)^{1/(a^r+b^r+c^r)}$

6. (Belarus MO) ให้ $\;a,b,c,x,y,z \in R^+\;$ จงพิสูจน์ว่า $\; \displaystyle{\frac{a^3}{x} + \frac{b^3}{y} + \frac{c^3}{z} \geqslant \frac{(a+b+c)^3}{3(x+y+z)}}$

โจทย์บทที่ 2 ครบหมดแล้ว ... โจทย์บทที่ 3 จะโพสต์เอาไว้ในความเห็น # 28 ครับ!
__________________
หนึ่งปีของอัจฉริยะ อาจเทียบเท่าชั่วชีวิตของคนบางคน

11 พฤษภาคม 2010 18:54 : ข้อความนี้ถูกแก้ไขแล้ว 10 ครั้ง, ครั้งล่าสุดโดยคุณ Switchgear
ตอบพร้อมอ้างอิงข้อความนี้
ตั้งหัวข้อใหม่ Reply


หัวข้อคล้ายคลึงกัน
หัวข้อ ผู้ตั้งหัวข้อ ห้อง คำตอบ ข้อความล่าสุด
"TU" or "MWIT" The jumpers ฟรีสไตล์ 28 26 สิงหาคม 2010 03:47
"Songkran" Festival Siren-Of-Step ฟรีสไตล์ 11 11 เมษายน 2010 20:29
Teach Me "Homothety" The jumpers เรขาคณิต 2 29 พฤศจิกายน 2009 21:33
กรุณา"แสดงวิธีทำ" 5 ข้อExpo&Log ให้ดูหน่อยครับ rattachin calculated ปัญหาคณิตศาสตร์ ม.ปลาย 11 14 พฤษภาคม 2009 15:45
ถึงพี่ "nongtum" comza ฟรีสไตล์ 1 09 มกราคม 2008 21:49


กฎการส่งข้อความ
คุณ ไม่สามารถ ตั้งหัวข้อใหม่ได้
คุณ ไม่สามารถ ตอบหัวข้อได้
คุณ ไม่สามารถ แนบไฟล์และเอกสารได้
คุณ ไม่สามารถ แก้ไขข้อความของคุณเองได้

vB code is On
Smilies are On
[IMG] code is On
HTML code is Off
ทางลัดสู่ห้อง


เวลาที่แสดงทั้งหมด เป็นเวลาที่ประเทศไทย (GMT +7) ขณะนี้เป็นเวลา 05:56


Powered by vBulletin® Copyright ©2000 - 2024, Jelsoft Enterprises Ltd.
Modified by Jetsada Karnpracha